Sie sind auf Seite 1von 16

Shear Stress compressive stress in the punch is limited to 50 ksi.

(a) Compute the maximum thickness of plate in


Forces parallel to the area resisting the force cause
which a hole 2.5 inches in diameter can be punched.
shearing stress. It differs to tensile and compressive
(b) If the plate is 0.25 inch thick, determine the
stresses, which are caused by forces perpendicular
diameter of the smallest hole that can be punched.
to the area on which they act. Shearing stress is also
known as tangential stress.

τ=VAτ=VA

Solution 116
where V is the resultant shearing force which passes
through the centroid of the area A being sheared. (a) Maximum thickness of plate:

Problem 115 Based on puncher strength:


What force is required to punch a 20-mm-diameter P=σA
hole in a plate that is 25 mm thick? The shear
P=50[14π(2.52)]
strength is 350 MN/m2.
P=78.125πkips → Equivalent shear force
of the plate
Solution 115

The resisting area is the shaded area along the


Based on shear strength of plate:
perimeter and the shear force VV is equal to the
V=τA → V=P
punching force PP.
78.125π=40[π(2.5t)]

t=0.781inch answer

(b) Diameter of smallest hole:

Based on compression of puncher:


P=σA

P=50(14πd2)

P=12.5πd2 → Equivalent shear force for plate

V=τAV=τA
Based on shearing of plate:
P=350[π(20)(25)] V=τ → V=P

P=549778.7N 12.5πd2=40[πd(0.25)]

P=549.8kN answer d=0.8in answer

Problem 116
As in Fig. 1-11c, a hole is to be punched out of a
plate having a shearing strength of 40 ksi. The
Problem 117
Find the smallest
ΣFH=0ΣFH=0
diameter bolt that can be
used in the clevis shown RBH=40cos35∘
in Fig. 1-11b if P =
400 kN. The RBH=32.766kN
shearing strength
of the bolt is 300
RB=RBH2+RBV2−−−−−−−−−−−√RB=RBH2+RBV2
MPa.
RB=32.7662+49.1562−−−−−−−−−−−−−−

RB=59.076kN → shear force of pin at B

VB=τBAVB=τBA → double shear

59.076(1000)=τB{2[14π(202)]}
Solution 117
τB=94.02 MPa answer
The bolt is subject to double shear.
V=τAV=τA

400(1000)=300[2(14πd2)]400(1000)=300[2(14πd2)]

d=29.13mmd=29.13mm answer Problem 120


The members of the structure in Fig. P-120 weigh
200 lb/ft. Determine the smallest diameter pin that
can be used at A if the shearing stress is limited to
5000 psi. Assume single shear.
Problem 119
Compute the shearing stress in the pin at B for the
member supported as shown in Fig. P-119. The pin
diameter is 20 mm.

Solution 120

For member AB:

Solution 119

HideClick here to show or hide the solution

From the FBD:


ΣMC=0ΣMC=0

0.25RBV=0.25(40sin35∘)+0.2(40cos35∘)

RBV=49.156kN
Length, LAB=42+42−−−−−−√=5.66ftLAB=42+42=5.66f RBV=153.67lbRBV=153.67lb
t

Weight, WAB=5.66(200)=1132lbWAB=5.66(200)=113 From the FBD of member AB


2lb ΣFH=0ΣFH=0

RAH=RBH=412.33lbRAH=RBH=412.33lb
ΣMA=0ΣMA=0

4RBH+4RBV=2WAB4RBH+4RBV=2WAB ΣFV=0ΣFV=0

4RBH+4RBV=2(1132)4RBH+4RBV=2(1132) RAV+RBV=WABRAV+RBV=WAB

RBH+RBV=566RBH+RBV=566 → Equation (1) RAV+153.67=1132RAV+153.67=1132

RAV=978.33lbRAV=978.33lb
For member BC:

RA=R2AH+R2AV−−−−−−−−−−√RA=RAH2+RAV2

RA=412.332+978.332−−−−−−−−−−−−−−√RA=412.332+
978.332

RA=1061.67lbRA=1061.67lb → shear force of pin


at A

V=τAV=τA
Length, LBC=√32+62=6.71ftLBC=32+62=6.71ft
1061.67=5000(14πd2)1061.67=5000(14πd2)
Weight, WBC=6.71(200)=WBC=1342lbWBC=6.71(200
)=WBC=1342lb d=0.520ind=0.520in answer

ΣMC=0ΣMC=0

6RBH=1.5WBC+3RBV6RBH=1.5WBC+3RBV
Bearing Stress
6RBH−3RBV=1.5(1342)6RBH−3RBV=1.5(1342)
Bearing stress is the contact pressure between
2RBH−RBV=6712RBH−RBV=671 → Equation (2)
the separate bodies. It differs from compressive
stress, as it is an internal stress caused by
Add equations (1) and (2)
RBH+RBV=566RBH+RBV=566 → Equation (1) compressive forces.

2RBH−RBV=6712RBH−RBV=671 → Equation (2)

3RBH=12373RBH=1237 σb=PbAb

RBH=412.33lbRBH=412.33lb

From equation (1):


Problem 125
412.33+RBV=566412.33+RBV=566
In Fig. 1-12, assume that a 20-mm-diameter
rivet joins the plates that are each 110 mm wide.
The allowable stresses are 120 MPa for bearing
in the plate material and 60 MPa for shearing of Solution 126
rivet. Determine (a) the minimum thickness of
each plate; and (b) the largest average tensile HideClick here to show or hide the solution
stress in the plates. Based on shearing of rivets:
P=τAP=τA
P=14[4(14π)(34)2]P=14[4(14π)(34)2]
P=24.74kipsP=24.74kips

Based on bearing of plates:


P=σbAbP=σbAb
P=18[4(34)(78)]P=18[4(34)(78)]
P=47.25kipsP=47.25kips

Safe load P,
P=24.74kipsP=24.74kips answer

Solution 125
Problem 127
Part (a): In the clevis shown in Fig. 1-11b, find the
From shearing of rivet: minimum bolt diameter and the minimum
P=τArivetsP=τArivets thickness of each yoke that will support a load P
= 14 kips without exceeding a shearing stress of
P=60[14π(202)]P=60[14π(202)] 12 ksi and a bearing stress of 20 ksi.
P=6000πNP=6000πN

From bearing of plate material:


P=σbAbP=σbAb
6000π=120(20t)6000π=120(20t)
t=7.85mmt=7.85mm answer

Part (b): Largest average tensile stress in the plate:


P=σAP=σA
6000π=σ[7.85(110−20)]6000π=σ[7.85(110−20)]
σ=26.67MPaσ=26.67MPa answer

Problem 126 Solution 127


The lap joint shown in Fig. P-126 is fastened by
four ¾-in.-diameter rivets. Calculate the For shearing of rivets (double shear)
maximum safe load P that can be applied if the P=τAP=τA
shearing stress in the rivets is limited to 14 ksi 14=12[2(14πd2)]14=12[2(14πd2)]
and the bearing stress in the plates is limited to d=0.8618ind=0.8618in → diameter
18 ksi. Assume the applied load is uniformly of bolt answer
distributed among the four rivets.
For bearing of yoke:
P=σbAbP=σbAb
14=20[2(0.8618t)]14=20[2(0.8618t)]
t=0.4061int=0.4061in → thickness
of yoke answer
Thin-walled Pressure Vessels σL=pD4tσL=pD4t

A tank or pipe carrying a fluid or gas under a


If there exist an external pressure po and an
pressure is subjected to tensile forces, which
internal pressure pi, the formula may be
resist bursting, developed across longitudinal expressed as:
and transverse sections. σL=(pi−po)D4tσL=(pi−po)D4t

The forces acting are the total pressures caused It can be observed that the tangential stress is
by the internal pressure p and the total tension twice that of the longitudinal stress.
in the walls T.
F=pA=pDLF=pA=pDL σt=2σLσt=2σL
T=σtAwall=σttLT=σtAwall=σttL

ΣFH=0ΣFH=0
F=2TF=2T
pDL=2(σttL)pDL=2(σttL)
σt=pD2tσt=pD2t

If there exist an external pressure po and an


internal pressure pi, the formula may be
expressed as:

σt=(pi−po)D2tσt=(pi−po)D2t

LONGITUDINAL STRESS, σL
Consider the free body diagram in the transverse
section of the tank:

The total force acting at the rear of the tank F


must equal to the total longitudinal stress on the
wall PT = σLAwall. Since t is so small compared to
D, the area of the wall is close to πDt
F=pA=pπ4D2F=pA=pπ4D2
PT=σLπDtPT=σLπDt

ΣFH=0ΣFH=0
PT=FPT=F
σLπDt=pπ4D2σLπDt=pπ4D2
Problem 133 Part (b)
A cylindrical steel pressure vessel 400 mm in
diameter with a wall thickness of 20 mm, is From
subjected to an internal pressure of 4.5 MN/m2. (a), σt=pD2t and σl=pD4t thus, σt=2σlσt=2σ
(a) Calculate the tangential and longitudinal l, this shows that tangential stress is the
stresses in the steel. (b) To what value may the critical.
internal pressure be increased if the stress in the
σt=pD2tσt=pD2t
steel is limited to 120 MN/m2? (c) If the internal
pressure were increased until the vessel burst, 120=p(400)2(20)120=p(400)2(20)
sketch the type of fracture that would occur. p=12MPap=12MPa answer

Solution 133 Problem 134


Part (a) The wall thickness of a 4-ft-diameter spherical
tank is 5/16 inch. Calculate the allowable
Tangential stress (longitudinal section): internal pressure if the stress is limited to 8000
psi.

Solution 134

F=2TF=2T
pDL=2(σttL)pDL=2(σttL)
σt=pD2t=4.5(400)2(20)σt=pD2t=4.5(400)2(
20)
σt=45MPaσt=45MPa answer
Total internal pressure:
Longitudinal Stress (transverse section): P=p(14πD2)P=p(14πD2)

Resisting wall:
F=PF=P
σA=p(14πD2)
σ(πDt)=p(14πD2)σ(πDt)=p(14πD2)
σ=pD4tσ=pD4t
8000=p(4×12)4(516)8000=p(4×12)4(516)
p=208.33psip=208.33psi answer

F=PF=P Problem 135


14πD2p=σl(πDt)14πD2p=σl(πDt) Calculate the minimum wall thickness for a
σl=pD4t=4.5(400)4(20)σl=pD4t=4.5(400)4( cylindrical vessel that is to carry a gas at a
20) pressure of 1400 psi. The diameter of the vessel
σl=22.5MPaσl=22.5MPa answer is 2 ft, and the stress is limited to 12 ksi.
Solution 135 Simple Strain

The critical stress is the tangential stress Also known as unit deformation, strain is the
σt=pD2tσt=pD2t
12000=1400(2×12)2t12000=1400(2×12)2t ratio of the change in length caused by the
t=1.4 in answer applied force, to the original length.

Problem 136
A cylindrical pressure vessel is fabricated from
steel plating that has a thickness of 20 mm. The
diameter of the pressure vessel is 450 mm and
its length is 2.0 m. Determine the maximum
internal pressure that can be applied if the ε=δLε=δL
longitudinal stress is limited to 140 MPa, and
the circumferential stress is limited to 60 MPa. where δ is the deformation and L is the original
length, thus ε is dimensionless.
Solution 136
Stress-strain Diagram
Based on circumferential stress (tangential):
σ=Eε

Axial Deformation

In the linear portion of the stress-strain


diagram, the tress is proportional to strain and is
given by
ΣFV=0ΣFV=0
F=2TF=2T
p(DL)=2(σtLt)p(DL)=2(σtLt)
σt=pD2tσt=pD2t σ=Eεσ=Eε
60=p(450)2(20)60=p(450)2(20)
p=5.33MPap=5.33MPa

Based on longitudinal stress: since σ=P/Aσ=P/A and ε=δ/Lε=δ/L,


then PA=EδLPA=EδL

δ=PLAE=σLE

Solution to Problem 205 Axial Deformation

ΣFH=0ΣFH=0 Problem 205


F=PF=P A uniform bar of length L, cross-sectional area
p(14πD2)=σl(πD)p(14πD2)=σl(πD)
σl=pD4tσl=pD4t A, and unit mass ρ is suspended vertically from
140=p(450)4(20)140=p(450)4(20)
one end. Show that its total elongation
p=24.89MPap=24.89MPa
is δ = ρgL2/2E. If the total mass of the bar is M,
Use p=5.33MPap=5.33MPa answer
Problem 206
show also that δ = MgL/2AE. A steel rod having a cross-sectional area of 300
mm2 and a length of 150 m is suspended
vertically from one end. It supports a tensile load
of 20 kN at the lower end. If the unit mass of
Solution 205 steel is 7850 kg/m3 and E = 200 × 103 MN/m2,
find the total elongation of the rod.
δ=PLAEδ=PLAE
From the figure: Solution 206
δ=dδδ=dδ
Elongation due to its own
P=Wy=(ρAy)gP=Wy=(ρAy)g weight:
δ1=PLAEδ1=PLAE
L=dyL=dy

Where:
P=W=
Thus,
dδ=(ρAy)gdyAEdδ=(ρAy)gdyAE
7850(1/1000)3(9.81)[300(150)(1000)]
δ=ρgE∫L0ydy=ρgE[y22]L0δ=ρgE∫0Lydy=ρg P = 3465.3825 N
E[y22]0L L = 75(1000) = 75 000 mm
A = 300 mm2
δ=ρg2E[L2−02]δ=ρg2E[L2−02] E = 200 000 MPa
δ=ρgL2/2Eδ=ρgL2/2E (okay!)
Thus,
δ1=3465.3825(75000)300(200000)δ1=3465.3825(75
Given the total mass M 000)300(200000)
ρ=M/V=M/ALρ=M/V=M/AL δ1=4.33 mmδ1=4.33 mm

δ=ρgL22E=MAL⋅gL22Eδ=ρgL22E=MAL⋅gL Elongation due to applied load:


δ2=PLAEδ2=PLAE
22E
δ=MgL2AEδ=MgL2AE (okay!) Where:
P = 20 kN = 20 000 N
L = 150 m = 150 000 mm
Another Solution: A = 300 mm2
δ=PLAEδ=PLAE E = 200 000 MPa

Thus,
Where: δ2=20000(150000)300(200000)δ2=20000(150000)30
P=W=(ρAL)gP=W=(ρAL)g 0(200000)
δ2=50 mmδ2=50 mm
L=12LL=12L
Total elongation:
δ=δ1+δ2δ=δ1+δ2
δ=4.33+50=54.33 mmδ=4.33+50=54.33 mm
Thus, answer
δ=[(ρAL)g](12L)AEδ=[(ρAL)g](12L)AE
δ=ρgL2/2Eδ=ρgL2/2E (okay!)
Problem 207 L = 1500π mm
A steel wire 30 ft long, hanging vertically, A = 10(80) = 800 mm2
supports a load of 500 lb. Neglecting the weight E = 200 000 MPa
of the wire, determine the required diameter if
the stress is not to exceed 20 ksi and the total
elongation is not to exceed 0.20 in. Assume E = Thus,
29 × 106 psi. 0.5π=T(1500π)800(200000)0.5π=T(1500π)800(2000
00)
T=53333.33NT=53333.33N
Solution 207

Based on maximum allowable stress:


σ=PAσ=PA
20000=50014πd220000=50014πd2
d=0.1784ind=0.1784in

Based on maximum allowable


deformation:
δ=PLAEδ=PLAE

0.20=500(30×12)14πd2(29×106)0.20=500(30×12)14 F=2TF=2T
πd2(29×106) p(1500)(80)=2(53333.33)p(1500)(80)=2(53333.33)
d=0.1988ind=0.1988in p=0.8889MPap=0.8889MPa → internal pressure

Use the bigger diameter, d = 0.1988 Total normal force, N:


inch. Answer N = p × contact area between tire and wheel
N = 0.8889 × π(1500.5)(80)
Problem 208 N = 335 214.92 N
A steel tire, 10 mm thick, 80 mm wide, and
1500.0 mm inside diameter, is heated and Friction resistance, f:
shrunk onto a steel wheel 1500.5 mm in f = μN = 0.30(335 214.92)
diameter. If the coefficient of static friction is f = 100 564.48 N = 100.56 kN
0.30, what torque is required to twist the tire Torque = f × ½(diameter of wheel)
relative to the wheel? Neglect the deformation of Torque = 100.56 × 0.75025
the wheel. Use E = 200 GPa. Torque = 75.44 kN · m

Problem 209
Solution 208 An aluminum bar having a cross-sectional area
of 0.5 in2 carries the axial loads applied at the
HideClick here to show or hide the solution positions shown in Fig. P-209. Compute the
total change in length of the bar if E = 10 ×
106 psi. Assume the bar is suitably braced to
prevent lateral buckling.

δ=PLAEδ=PLAE
Solution 209

Where: HideClick here to show or hide the solution


δ = π (1500.5 - 1500) = 0.5π mm P1 = 6000 lb tension
P=T P2 = 1000 lb compression
P3 = 4000 lb tension Solution to Problem 225 Biaxial Deformation

Problem 225
A welded steel cylindrical drum made of a 10-
mm plate has an internal diameter of 1.20 m.
Compute the change in diameter that would be
caused by an internal pressure of 1.5 MPa.
Assume that Poisson's ratio is 0.30 and E = 200
GPa.
δ=PLAEδ=PLAE

δ=δ1−δ2+δ3δ=δ1−δ2+δ3
δ=6000(3×12)0.5(10×106)−1000(5×12)0.5(10×106)+ Solution 225
4000(4×12)0.5(10×106)δ=6000(3×12)0.5(10×106)−1
000(5×12)0.5(10×106)+4000(4×12)0.5(10×106)
σy = longitudinal stress
δ=0.0696 in. (lengthening)δ=0.0696 in.
(lengthening) answer σy=pD4t=1.5(1200)4(10)σy=pD4t=1.5(1200)4(10)
σy=45MPaσy=45MPa
Solution to Problem 222 Poisson's Ratio

Problem 222 σx = tangential stress

A solid cylinder of diameter d carries an axial σy=pD2t=1.5(1200)2(10)σy=pD2t=1.5(1200)2(10)

load P. Show that its change in diameter is σy=90MPaσy=90MPa

4Pν / πEd.

Solution 222

εx=σxE−νσyEεx=σxE−νσyE
εx=90200000−0.3(45200000)εx=90200000−0.3(4
ν=−εyεxν=−εyεx
5200000)
εy=−νεxεy=−νεx
εx=3.825×10−4εx=3.825×10−4
εy=−νσxEεy=−νσxE
δyd=−ν−PAEδyd=−ν−PAE
εx=ΔDDεx=ΔDD
δy=Pd14πd2Eδy=Pd14πd2E
ΔD=εxD=(3.825×10−4)(1200)ΔD=εxD=(3.825×10
δy=4PνπEdδy=4PνπEd (okay!)
−4)(1200)
ΔD=0.459 mmΔD=0.459 mm answer
Solution to Problem 226 Biaxial Deformation = 100 GPa.

Problem 226
Solution 233
A 2-in.-diameter steel tube with a wall thickness
δ=PLAEδ=PLAE
of 0.05 inch just fits in a rigid hole. Find the δ=δcastiron=δsteel=0.8mmδ=δcastiron=δsteel=0.8m
tangential stress if an axial compressive load of m
3140 lb is applied. Assume ν = 0.30 and neglect δcastiron=Pcastiron(2000)[14π(602−502)](100000)=
the possibility of buckling. 0.8δcastiron=Pcastiron(2000)[14π(602−502)](100000
)=0.8
Pcastiron=11000πNPcastiron=11000πN

Solution 226

δsteel=Psteel(2000)[14π(502)](200000)=0.8δsteel=Ps
teel(2000)[14π(502)](200000)=0.8
εx=σxE−νσyE=0εx=σxE−νσyE=0 Psteel=50000πNPsteel=50000πN
σx=νσyσx=νσy
ΣFV=0ΣFV=0
P=Pcastiron+PsteelP=Pcastiron+Psteel
P=11000π+50000πP=11000π+50000π
Where P=61000πNP=61000πN
P=191.64kNP=191.64kN answer
σx = tangential stress
σy = longitudinal stress
Solution to Problem 234 Statically Indeterminate
σy = Py / A = 3140 / (π × 2 × 0.05)
σy = 31,400/π psi Problem 234
A reinforced concrete column 200 mm in
diameter is designed to carry an axial
Thus,
compressive load of 300 kN. Determine the
σx=0.30(31400/π)σx=0.30(31400/π)
required area of the reinforcing steel if the
σx=9430/π psiσx=9430/π psi
allowable stresses are 6 MPa and 120 MPa for
σx=2298.5 psiσx=2298.5 psi answer
the concrete and steel, respectively. Use Eco = 14
GPa and Est = 200 GPa.
Problem 233
A steel bar 50 mm in diameter and 2 m long is
surrounded by a shell of a cast iron 5 mm thick.
Compute the load that will compress the
combined bar a total of 0.8 mm in the length of
2 m. For steel, E = 200 GPa, and for cast iron, E
Problem 235
Solution 234 A timber column, 8 in. × 8 in. in cross section, is
reinforced on each side by a steel plate 8 in. wide
δco=δst=δδco=δst=δ and t in. thick. Determine the thickness t so that
the column will support an axial load of 300 kips
without exceeding a maximum timber stress of
(PLAE)co=(PLAE)st(PLAE)co=(PLAE)st
1200 psi or a maximum steel stress of 20 ksi.
(σLE)co=(σLE)st(σLE)co=(σLE)st The moduli of elasticity are 1.5 × 106 psi for
timber, and 29 × 106 psi for steel.
σcoL14000=σstL200000σcoL14000=σstL200000
100σco=7σst100σco=7σst
Solution 235

HideClick here to show or hide the solution


When σst = 120 MPa δsteel=δtimberδsteel=δtimber
100σco=7(120)100σco=7(120)

σco=8.4 MPa>6 MPaσco=8.4 MPa>6 MPa →


(not okay!)
(σLE)steel=(σLE)timber(σLE)steel=(σLE)timber
σsteelL29×106=σbronzeL1.5×106σsteelL29×106=σb
When σco = 6 MPa ronzeL1.5×106
100(6)=7σst100(6)=7σst 1.5σsteel=29σtimber1.5σsteel=29σtimber

σst=85.71 MPa<120 MPaσst=85.71 MPa<120 MPa When σtimber = 1200 psi


→ (okay!) 1.5σsteel=29(1200)1.5σsteel=29(1200)
σsteel=23200 psi =23.2 ksi >20 ksi σsteel=23200 psi
=23.2 ksi >20 ksi (not ok!)

Use σco = 6 MPa and σst = 85.71 MPa. When σsteel = 20 ksi
1.5(20×1000)=29σtimber1.5(20×1000)=29σtimber
$\sigma_{timber} = 1034.48 \, \text{ psi }
ΣFV=0ΣFV=0 Use σsteel = 20 ksi and σtimber = 1.03 ksi
Pst+Pco=300Pst+Pco=300
σstAst+σcoAco=300σstAst+σcoAco=300 ΣFV=0ΣFV=0
Fsteel+Ftimber=300Fsteel+Ftimber=300
85.71Ast+6[14π(200)2−Ast]=300(1000)85.71Ast+6[
(σA)steel+(σA)timber=300(σA)steel+(σA)timber=30
14π(200)2−Ast]=300(1000) 0
20[4(8t)]+1.03(82)=30020[4(8t)]+1.03(82)=300
79.71Ast+60000π=30000079.71Ast+60000π=300 t=0.365 int=0.365 in answer
000
Ast=1398.9 mm2Ast=1398.9 mm2 answer
Problem 236 Solution to Problem 261 Thermal Stress
A rigid block of mass M is supported by three
symmetrically spaced rods as shown in Fig. P- Problem 261
236. Each copper rod has an area of 900 mm2; E
= 120 GPa; and the allowable stress is 70 MPa. A steel rod with a cross-sectional area of 0.25
The steel rod has an area of 1200 mm2; E = 200 in2 is stretched between two fixed points. The
GPa; and the allowable stress is 140 MPa.
Determine the largest mass M which can be tensile load at 70°F is 1200 lb. What will be the
supported.
stress at 0°F? At what temperature will the
stress be zero? Assume α = 6.5 × 10-6 in/(in·°F)
and E = 29 × 106 psi.

Solution 261

For the stress at 0°F:

Solution 236

δ=δT+δstδ=δT+δst
σLE=αL(ΔT)+PLAEσLE=αL(ΔT)+PLAE
σ=αE(ΔT)+PAσ=αE(ΔT)+PA
δco=δstδco=δst σ=(6.5×10−6)(29×106)(70)+12000.25σ=(6.
(σLE)co=(σLE)st(σLE)co=(σLE)st
5×10−6)(29×106)(70)+12000.25
σco(160)120000=σst(240)200000σco(160)120000=σ
st(240)200000 σ=17995psi=18 ksiσ=17995psi=18 ksi
10σco=9σst10σco=9σst
answer
When σst = 140 MPa
σco=910(140)σco=910(140)
σco=126 MPa >70 MPa σco=126 MPa >70 MPa (no
t okay!) For the temperature that causes zero stress:

When σco = 70 MPa


σst=109(70)σst=109(70)
σst=77.78 MPa <140 MPa σst=77.78 MPa <140 MPa
(okay!)

Use σco = 70 MPa and σst = 77.78 MPa.

ΣFV=0ΣFV=0
2Pco+Pst=W2Pco+Pst=W
2(σcoAco)+σstAst=Mg2(σcoAco)+σstAst=Mg
2[70(900)]+77.78(1200)=M(9.81)2[70(900)]+77.78(
1200)=M(9.81)
M=22358.4 kgM=22358.4 kg answer δT=δstδT=δst
αL(ΔT)=PLAEαL(ΔT)=PLAE
α(ΔT)=PAEα(ΔT)=PAE
(6.5×10−6)(T−70)=12000.25(29×106)(6.5×
10−6)(T−70)=12000.25(29×106)
T=95.46∘FT=95.46∘F answer
Temperature at which δT = 3 mm:
δT=αL(ΔT)δT=αL(ΔT)
Problem 262 δT=αL(Tf−Ti)δT=αL(Tf−Ti)
A steel rod is stretched between two rigid walls 3=(11.7×10−6)(10000)(Tf−15)3=(11.7×10−6)(10000
and carries a tensile load of 5000 N at 20°C. If )(Tf−15)
the allowable stress is not to exceed 130 MPa at - Tf=40.64∘CTf=40.64∘C answer
20°C, what is the minimum diameter of the rod?
Assume α = 11.7 µm/(m·°C) and E = 200 GPa. Required stress:
δ=δTδ=δT
Solution 262 σLE=αL(ΔT)σLE=αL(ΔT)
σ=αE(Tf−Ti)σ=αE(Tf−Ti)
HideClick here to show or hide the solution σ=(11.7×10−6)(200000)(40.64−15)σ=(11.7×10−6)(2
00000)(40.64−15)
σ=60 MPaσ=60 MPa answer

Problem 264
A steel rod 3 feet long with a cross-sectional area
of 0.25 in.2 is stretched between two fixed
points. The tensile force is 1200 lb at 40°F.
Using E = 29 × 106 psi and α = 6.5 × 10-
6 in./(in.·°F), calculate (a) the temperature at
δ=δT+δstδ=δT+δst
σLE=αL(ΔT)+PLAEσLE=αL(ΔT)+PLAE which the stress in the bar will be 10 ksi; and (b)
σ=αE(ΔT)+PAσ=αE(ΔT)+PA the temperature at which the stress will be zero.
130=(11.7×10−6)(200000)(40)+5000A130=(11.7×10
−6)(200000)(40)+5000A
Solution 264
A=500036.4=137.36 mm2A=500036.4=137.36 mm2
HideClick here to show or hide the solution
14πd2=137.3614πd2=137.36
d=13.22 mmd=13.22 mm answer (a) Without temperature change:

σ=PA=12000.25=4800 psi σ=PA=12000.25=


Problem 263 4800 psi
Steel railroad reels 10 m long are laid with a
clearance of 3 mm at a temperature of 15°C. At σ=4.8ksi<10ksiσ=4.8ksi<10ksi
what temperature will the rails just touch? What A drop of temperature is needed to increase
stress would be induced in the rails at that the stress to 10 ksi. See figure below.
temperature if there were no initial clearance?
Assume α = 11.7 µm/(m·°C) and E = 200 GPa.

Solution 263

HideClick here to show or hide the solution

δ=δT+δstδ=δT+δst
σLE=αL(ΔT)+PLAEσLE=αL(ΔT)+PLAE
σ=αE(ΔT)+PAσ=αE(ΔT)+PA τmax=14.3ksiτmax=14.3ksi answer
10000=(6.5×10−6)(29×106)(ΔT)+12000.25
10000=(6.5×10−6)(29×106)(ΔT)+12000.25
ΔT=27.59∘FΔT=27.59∘F θ=TLJG=15(3)(1000)(122)132π(44)(12×106)θ=TLJ
G=15(3)(1000)(122)132π(44)(12×106)
Required temperature: (temperature must
drop from 40°F) θ=0.0215radθ=0.0215rad
T=40−27.59=12.41∘FT=40−27.59=12.41∘F θ=1.23∘θ=1.23∘ answer
answer

Problem 305
(b) Temperature at which the stress will be What is the minimum diameter of a solid steel
zero: shaft that will not twist through more than 3° in
a 6-m length when subjected to a torque of 12
From the figure below: kN·m? What maximum shearing stress is
developed? Use G = 83 GPa.

Solution 305

θ=TLJGθ=TLJG
3∘(π180∘)=12(6)(10003)132πd4(83000)3∘(π180∘)=1
2(6)(10003)132πd4(83000)
d=113.98mmd=113.98mm answer

τmax=16Tπd3=16(12)(10002)π(113.983)τmax=16Tπ
δ=δTδ=δT d3=16(12)(10002)π(113.983)
PLAE=αL(ΔT)PLAE=αL(ΔT) τmax=41.27MPaτmax=41.27MPa answer
P=αAE(Tf−Ti)P=αAE(Tf−Ti)
1200=(6.5×10−6)(0.25)(29×106)(Tf−40)120
Problem 306
0=(6.5×10−6)(0.25)(29×106)(Tf−40)
A steel marine propeller shaft 14 in. in diameter
Tf=65.46∘FTf=65.46∘F answer and 18 ft long is used to transmit 5000 hp at 189
rpm. If G = 12 × 106 psi, determine the
maximum shearing stress.
Solution to Problem 304 Torsion

Problem 304 Solution 306


A steel shaft 3 ft long that has a diameter of 4 in
HideClick here to show or hide the solution
is subjected to a torque of 15 kip·ft. Determine Note: 1 hp = 33,000 ft·lb/min = 396,000 lb·in/min
T=P2πf=5000(396000)2π(189)T=P2πf=5000(396000
the maximum shearing stress and the angle of )2π(189)
twist. Use G = 12 × 106 psi. T=1667337.5lb⋅inT=1667337.5lb⋅in

τmax=16Tπd3=16(1667337.5)π(143)τmax=16Tπd3=
16(1667337.5)π(143)
Solution 304 τmax=3094.6psiτmax=3094.6psi answer

τmax=16TπD3=16(15)(1000)(12)π(43)τmax=16Tπ Problem 307


D3=16(15)(1000)(12)π(43) A solid steel shaft 5 m long is stressed at 80 MPa
when twisted through 4°. Using G = 83 GPa,
τmax=14324psiτmax=14324psi compute the shaft diameter. What power can be
transmitted by the shaft at 20 Hz?

Solution 307

θ=TLJGθ=TLJG
4∘(π180∘)=T(5)(1000)132πd4(83000)4∘(π180∘)=T(5
)(1000)132πd4(83000)
T=0.1138d4T=0.1138d4

τmax=16Tπd3τmax=16Tπd3
80=16(0.1138d4)πd380=16(0.1138d4)πd3
d=138 mmd=138 mm answer

T=P2πfT=P2πf
0.1138d4=P2π(20)0.1138d4=P2π(20)
P=14.3d4=14.3(1384)P=14.3d4=14.3(1384)
P=5186237285N⋅mm/secP=5186237285N⋅mm/sec
P=5186237.28WP=5186237.28W
P=5.19 MWP=5.19 MW answer

Problem 308
A 2-in-diameter steel shaft rotates at
240 rpm. If the shearing stress is limited
to 12 ksi, determine the maximum
horsepower that can be transmitted.

Solution 308
HideClick here to show or hide the solution
τmax=16Tπd3τmax=16Tπd3
12(1000)=16Tπ(23)12(1000)=16Tπ(23)
T=18849.56lb⋅inT=18849.56lb⋅in

T=P2πfT=P2πf
18849.56=P(396000)2π(240)18849.56=
P(396000)2π(240)
P=71.78 hpP=71.78 hp answer

Das könnte Ihnen auch gefallen